Difference between revisions of "2022 AMC 10A Problems/Problem 19"

(tex)
(Solution)
Line 34: Line 34:
  
 
~KingRavi
 
~KingRavi
 +
 
+mathboy282(tex)
 
+mathboy282(tex)
  

Revision as of 14:42, 12 November 2022

Problem

Define $L_n$ as the least common multiple of all the integers from $1$ to $n$ inclusive. There is a unique integer $h$ such that

$\frac{1}{1}+\frac{1}{2}+\frac{1}{3}\ldots+\frac{1}{17}=\frac{h}{L_{17}}$

What is the remainder when $h$ is divided by $17$?

$\textbf{(A) } 1 \qquad \textbf{(B) } 3 \qquad \textbf{(C) } 5 \qquad \textbf{(D) } 7 \qquad \textbf{(E) } 9$

Solution

Notice that $L_{17}$ contains the highest power of every prime below $17$. Thus, $L_{17}=16\cdot 9 \cdot 5 \cdot 7 \cdot 11 \cdot 13 \cdot 17$.

When writing the sum under a common fraction, we multiply the denominators by $L_{17}$ divided by each denominator. However, since $L_{17}$ is a multiple of $17$, all terms will be a multiple of $17$ until we divide out $17$, and the only term that will do this is $\frac{1}{17}$. Thus, the remainder of all other terms when divided by $17$ will be $0$, so the problem is essentially asking us what the remainder of $\frac{L_{17}}{17}$ divided by $17$ is. This is equivalent to finding the remainder of $16 \cdot 9 \cdot 5 \cdot 7 \cdot 11 \cdot 13$ divided by $17$.

We use modular arithmetic to simplify our answer:

This is congruent to $-1 \cdot 9 \cdot 5 \cdot 7 \cdot 11 \cdot 13 \pmod{17}$

Evaluating, we get: \begin{align*} -45 \cdot 7 \cdot 11 \cdot 13 \pmod{17} &\equiv 6 \cdot 7 \cdot 11 \cdot 13 \pmod{17} \\ &\equiv 42 \cdot 11 \cdot 13 \pmod{17} \\ &\equiv 8 \cdot 11 \cdot 13 \pmod{17} \\ &\equiv 88 \cdot 13 \pmod{17} \\ &\equiv 3 \cdot 13 \pmod{17} \\ &\equiv 39 \pmod{17} \\ &\equiv 5\pmod{17} \end{align*}

Therefore the remainder is $5$ and the answer is $\boxed{C}$


~KingRavi

+mathboy282(tex)

Video Solution By ThePuzzlr

https://youtu.be/TGcGamPXdNc

~ MathIsChess

See Also

2022 AMC 10A (ProblemsAnswer KeyResources)
Preceded by
Problem 18
Followed by
Problem 20
1 2 3 4 5 6 7 8 9 10 11 12 13 14 15 16 17 18 19 20 21 22 23 24 25
All AMC 10 Problems and Solutions

The problems on this page are copyrighted by the Mathematical Association of America's American Mathematics Competitions. AMC logo.png